Absolute Value Signs, Inequalities and Mechanics (1 Viewer)

frog1944

Member
Joined
Apr 9, 2016
Messages
210
Gender
Undisclosed
HSC
2017
Hi,

Context: When looking at the 2007 HSC Q 3 d) ii) where they asked for what values of w is N >), the MANSW solutions obtain that w^2 < g/rtan(Theta), and then state |w| < (g/rtan(theta))^0.5 . However, for the 2016 HSC Q 13 c) ii) they asked for what range of values of w is T_2 > T_1, their solutions state |w| > (200/3)^0.5) and then say w > 0, thus w > (200/3)^0.5 (though I couldn't find in the question that w>0)

Question: Is there a discrepancy between the different tests and their solutions? Or could someone please explain why for the 2016 w>0?

Thanks
 

raz999

New Member
Joined
Jun 25, 2016
Messages
9
Gender
Male
HSC
2017
The truth is in circular motion, the angular 'velocity' w is really angular speed, i.e. w is ALWAYS defined to be > 0. Hence |w| makes no difference.
 

frog1944

Member
Joined
Apr 9, 2016
Messages
210
Gender
Undisclosed
HSC
2017
Ok, so in the HSC I should always just take the positive root of w, as w is always > 0?
 

Users Who Are Viewing This Thread (Users: 0, Guests: 1)

Top